Difference between revisions of "2019 AMC 12A Problems/Problem 25"
Soyamyboya (talk | contribs) (→Problem) |
Soyamyboya (talk | contribs) (→Solution) |
||
Line 24: | Line 24: | ||
\end{align*}</cmath> | \end{align*}</cmath> | ||
− | Our induction is complete, so for all positive integers <math>n</math>, <math>x_n=(-2)^n(x_0-60)+60</math>. Identical equalities hold for <math>y_n</math> and <math>z_n</math> | + | Our induction is complete, so for all positive integers <math>n</math>, <math>x_n=(-2)^n(x_0-60)+60</math>. Identical equalities hold for <math>y_n</math> and <math>z_n</math>. |
The problem asks for the smallest <math>n</math> such that either <math>x_n</math>, <math>y_n</math>, or <math>z_n</math> is greater than <math>90^\circ</math>. WLOG, let <math>x_0=60^\circ</math>, <math>y_0=59.999^\circ</math>, and <math>z_0=60.001^\circ</math>. Thus, <math>x_n=60^\circ</math> for all <math>n</math>, <math>y_n=-(-2)^n(0.001)+60</math>, and <math>z_n=(-2)^n(0.001)+60</math>. Solving for the smallest possible value of <math>n</math> in each sequence, we find that <math>n=15</math> gives <math>y_n>90^\circ</math>. Therefore, the answer is <math>\boxed{\bold{(E) 15}}</math>. | The problem asks for the smallest <math>n</math> such that either <math>x_n</math>, <math>y_n</math>, or <math>z_n</math> is greater than <math>90^\circ</math>. WLOG, let <math>x_0=60^\circ</math>, <math>y_0=59.999^\circ</math>, and <math>z_0=60.001^\circ</math>. Thus, <math>x_n=60^\circ</math> for all <math>n</math>, <math>y_n=-(-2)^n(0.001)+60</math>, and <math>z_n=(-2)^n(0.001)+60</math>. Solving for the smallest possible value of <math>n</math> in each sequence, we find that <math>n=15</math> gives <math>y_n>90^\circ</math>. Therefore, the answer is <math>\boxed{\bold{(E) 15}}</math>. |
Revision as of 21:59, 9 February 2019
Problem
Let be a triangle whose angle measures are exactly , , and . For each positive integer define to be the foot of the altitude from to line . Likewise, define to be the foot of the altitude from to line , and to be the foot of the altitude from to line . What is the least positive integer for which is obtuse?
Solution
For all nonnegative integers , let , , and .
Note that quadrilateral is cyclic since ; thus, . By a similar argument, . Thus, . By a symmetric argument, and .
Therefore, for any positive integer , we have (identical recurrence relations can be derived for and ). To find an explicit form for this recurrence, we guess that the constant term is related exponentially to (and the coefficient of is ). Hence, we let . We will solve for , , and by iterating the recurrence to obtain , , and . Letting respectively, we have
Subtracting from , we have , and subtracting from gives . Dividing these two equations gives , so . Substituting back, we get and .
We will now prove that for all positive integers , via induction. Clearly the base case of holds, so it is left to prove that assuming our inductive hypothesis holds for . Using the recurrence relation, we have
Our induction is complete, so for all positive integers , . Identical equalities hold for and .
The problem asks for the smallest such that either , , or is greater than . WLOG, let , , and . Thus, for all , , and . Solving for the smallest possible value of in each sequence, we find that gives . Therefore, the answer is .
-soyamyboya
See Also
2019 AMC 12A (Problems • Answer Key • Resources) | |
Preceded by Problem 24 |
Followed by Last Problem |
1 • 2 • 3 • 4 • 5 • 6 • 7 • 8 • 9 • 10 • 11 • 12 • 13 • 14 • 15 • 16 • 17 • 18 • 19 • 20 • 21 • 22 • 23 • 24 • 25 | |
All AMC 12 Problems and Solutions |
The problems on this page are copyrighted by the Mathematical Association of America's American Mathematics Competitions.